Difference between revisions of "2000 AIME II Problems/Problem 3"
I like pie (talk | contribs) |
(rewrite) |
||
Line 1: | Line 1: | ||
== Problem == | == Problem == | ||
− | A deck of forty cards consists of four 1's, four 2's,..., and four 10's. A matching pair (two cards with the same number) is removed from the deck. Given that these cards are not returned to the deck, let <math>m/n</math> be the probability that two randomly selected cards also form a pair, where <math>m</math> and <math>n</math> are relatively prime positive integers. Find <math>m + n.</math> | + | A deck of forty cards consists of four <math>1</math>'s, four <math>2</math>'s,..., and four <math>10</math>'s. A matching pair (two cards with the same number) is removed from the deck. Given that these cards are not returned to the deck, let <math>m/n</math> be the [[probability]] that two randomly selected cards also form a pair, where <math>m</math> and <math>n</math> are [[relatively prime]] positive integers. Find <math>m + n.</math> |
== Solution == | == Solution == | ||
− | + | There are <math>{38 \choose 2} = 703</math> ways we can draw a two cards from the deck. The two cards will form a pair if both are one of the nine numbers that were not removed, which can happen in <math>9{4 \choose 2} = 54</math> ways, or if the two cards are the remaining two cards of the number that was removed, which can happen in <math>1</math> way. Thus, the answer is <math>\frac{54+1}{703} = \frac{55}{703}</math>, and <math>m+n = \boxed{758}</math>. | |
− | |||
− | The | ||
+ | == See also == | ||
{{AIME box|year=2000|n=II|num-b=2|num-a=4}} | {{AIME box|year=2000|n=II|num-b=2|num-a=4}} | ||
+ | [[Category:Intermediate Combinatorics Problems]] | ||
[[Category:Intermediate Probability Problems]] | [[Category:Intermediate Probability Problems]] |
Revision as of 10:31, 6 September 2008
Problem
A deck of forty cards consists of four 's, four 's,..., and four 's. A matching pair (two cards with the same number) is removed from the deck. Given that these cards are not returned to the deck, let be the probability that two randomly selected cards also form a pair, where and are relatively prime positive integers. Find
Solution
There are ways we can draw a two cards from the deck. The two cards will form a pair if both are one of the nine numbers that were not removed, which can happen in ways, or if the two cards are the remaining two cards of the number that was removed, which can happen in way. Thus, the answer is , and .
See also
2000 AIME II (Problems • Answer Key • Resources) | ||
Preceded by Problem 2 |
Followed by Problem 4 | |
1 • 2 • 3 • 4 • 5 • 6 • 7 • 8 • 9 • 10 • 11 • 12 • 13 • 14 • 15 | ||
All AIME Problems and Solutions |